Menu Close

Category: None

0-pi-sinx-2n-dx-n-N-

Question Number 144174 by lapache last updated on 22/Jun/21 $$\int_{\mathrm{0}} ^{\pi} \left({sinx}\right)^{\mathrm{2}{n}} {dx}=….?\:\:\:\forall{n}\in\mathbb{N} \\ $$ Commented by Willson last updated on 22/Jun/21 $$\mathrm{4}^{\mathrm{n}} \mathrm{sin}^{\mathrm{2n}} \mathrm{t}\:=\:\mathrm{C}_{\mathrm{2n}}…

I-e-x-2-e-x-e-x-2-1-dx-

Question Number 144152 by SOMEDAVONG last updated on 22/Jun/21 $$\mathrm{I}=\int\frac{\mathrm{e}^{\mathrm{x}^{\mathrm{2}} } +\mathrm{e}^{\mathrm{x}} }{\mathrm{e}^{\mathrm{x}^{\mathrm{2}} } +\mathrm{1}}\mathrm{dx}=? \\ $$ Answered by ArielVyny last updated on 22/Jun/21 $$=\int\frac{{e}^{{x}^{\mathrm{2}}…

A-lim-n-1-2-1-7-3-1-7-4-1-7-n-1-7-n-9-1-7-

Question Number 144107 by SOMEDAVONG last updated on 21/Jun/21 $$\mathrm{A}=\underset{\mathrm{n}\rightarrow+\propto} {\mathrm{lim}}\frac{\mathrm{1}+\sqrt[{\mathrm{7}}]{\mathrm{2}}+\sqrt[{\mathrm{7}}]{\mathrm{3}}+\sqrt[{\mathrm{7}}]{\mathrm{4}}+…..+\sqrt[{\mathrm{7}}]{\mathrm{n}}}{\:\sqrt[{\mathrm{7}}]{\mathrm{n}^{\mathrm{9}} }}\:=? \\ $$ Answered by Dwaipayan Shikari last updated on 21/Jun/21 $${A}=\underset{{n}\rightarrow\infty} {\mathrm{lim}}\frac{\mathrm{1}+\sqrt[{\mathrm{7}}]{\mathrm{2}}+..+\sqrt[{\mathrm{7}}]{{n}}}{\:\sqrt[{\mathrm{7}}]{{n}^{\mathrm{8}} }}…

Question-13030

Question Number 13030 by 2002154006 last updated on 11/May/17 Commented by prakash jain last updated on 12/May/17 $${x}^{\mathrm{2}} −\mathrm{4}{x}+\mathrm{5}=\mathrm{0} \\ $$$$\mathrm{x}_{\mathrm{1}} +\mathrm{x}_{\mathrm{2}} =\mathrm{4} \\ $$$$\mathrm{x}_{\mathrm{1}}…

Let-d-dx-F-x-e-sin-x-x-x-gt-0-If-1-4-2-e-sin-x-2-x-dx-F-k-F-1-then-one-of-the-possible-values-of-k-is-

Question Number 65961 by mmkkmm000m last updated on 06/Aug/19 $$\mathrm{Let}\:\frac{{d}}{{dx}}\left({F}\left({x}\right)\right)\:=\:\frac{{e}^{\mathrm{sin}\:{x}} }{{x}}\:,\:{x}>\mathrm{0}. \\ $$$$\mathrm{If}\:\underset{\:\mathrm{1}} {\overset{\mathrm{4}} {\int}}\:\:\frac{\mathrm{2}\:{e}^{\mathrm{sin}\:{x}^{\mathrm{2}} } }{{x}}\:{dx}\:=\:{F}\left({k}\right)−{F}\left(\mathrm{1}\right),\:\mathrm{then}\:\mathrm{one} \\ $$$$\mathrm{of}\:\mathrm{the}\:\mathrm{possible}\:\mathrm{values}\:\mathrm{of}\:\:{k}\:\:\mathrm{is} \\ $$ Answered by mr W…